《QUESTION 1》
The length of the side of a square is 50 cm and the breadth of a rectangle is 25 cm. If the areas of the square and the rectangle are equal, find the length of the rectangle.

《QUESTION 2》
A wire is in the shape of a rectangle with length 6 cm. It is bent to form a square whose side is 5 cm. Find the breadth of the rectangle. Also, calculate the areas of both the rectangle and the square and find which shape encloses more areas.​

Answers

Answer 1

The length of the rectangle is 100cm.

The breadth of the rectangle is 4cm.

The square encloses more area than the rectangle.

What is the length of the rectangle?

The first step is to determine the area of the square.

Area of a square = length² = 50² = 2500 cm²

Area of a rectangle = length x width

length = area / width

2500 / 25 = 100cm

What is the breadth of the rectangle?

The first step is to determine the perimeter of the square.

Perimeter = 4 x length

4 x 5 = 20cm

Perimeter of a rectangle = 2(length + width)

breadth = (perimeter / 2) - length

(20 / 2) - 6

10 - 6 = 4cm

Area of the square = 5 x 5 = 25

Area of the rectangle = 6 x 4 = 24

To learn more about the perimeter of a rectangle, please check: https://brainly.com/question/3205029

#SPJ1


Related Questions

O
11
O
12 13

14
15
Which number line represents the solutions to -2|x| = -6?

16
17
18

+ +
-8-7-6-5-4-3 -2 -1 0 1 2 3
+
-8-7-6-5-4-3 -2 -1 0 1
19
+
-8-7-6-5-4-3 -2 -1 0 1 2
2 3
++
20
++ ++
4
567
4 5 6 7
3 4
5 6 7
8
8
+
8
H
-8-7-6 -5 -4 -3 -2 -1 0 1 2 3 4 5 6 7 8
T

Answers

Answer:

thats not even understandable

The solution set is {3, -3} and is represented by the number line shown below.

What is number line?

It is the representation of numbers in real order.

The difference between the consecutive numbers in a number line is always positive.

We have,

To solve the equation -2|x| = -6,

We can begin by dividing both sides by -2:

|x| = 3

This means that the absolute value of x is equal to 3.

The solutions to this equation are x = 3 and x = -3.

The number line that represents these solutions is:

<--(+)===================(+)--->

   -3                                       3

The open circles indicate that the solutions are not included in the solution set since the absolute value of x cannot be negative.

Therefore,

The solution set is {3, -3} and is represented by the number line shown above.

Learn more about number line here:

https://brainly.com/question/13425491

#SPJ5

6/64 reduce to lowest terms


Answers

Answer:

[tex]\frac{3}{32}[/tex]

Step-by-step explanation:

-> Simplify

[tex]\frac{6}{64} =\frac{6/2}{64/2} =\frac{3}{32}[/tex]

6/64 simplified.

Find the zeros of the quadratic polynomial f(x) = 6x²-3, and verify the relationship between the zeros and its coefficients.​

Answers

Step-by-step explanation:

1) zeros of the given function:

6x²-3=0; ⇔ 6(x²-0.5)=0; ⇔ x²=0.5; ⇔

[tex]\left[\begin{array}{ccc}x=-\sqrt{0.5} \\x=\sqrt{0.5} \end{array}[/tex]

2) relationship:

if to see the equation x²-0.5=0 (ax²+bx+c=0 is standart form!), then the sum of the zeros is '0' (it is 'b' of the standart form), the product of equation roots is '-0.5' (it is 'c' of the standart form).

[tex]{\large{\textsf{\textbf{\underline{\underline{Given :}}}}}}[/tex]

★ The polynomial

f(x) = 6x² - 3

[tex]{\large{\textsf{\textbf{\underline{\underline{To \: find :}}}}}}[/tex]

★ Zeroes of the polynomial f(x) = 6x² - 3

[tex]{\large{\textsf{\textbf{\underline{\underline{Solution :}}}}}}[/tex]

We have,

[tex]f(x) = \tt 6 {x}^{2} - 3[/tex]

Which can also be written as

[tex] \implies f(x) = \tt {(\sqrt{6} x)}^{2} - { (\sqrt{3}) }^{2} [/tex]

Using a² - b² = (a - b) (a + b)

[tex] \implies f(x) = \tt ( \sqrt{6} x - \sqrt{3} )( \sqrt{6} x + \sqrt{3} )[/tex]

To find the zeroes, solve f(x) = 0

[tex] \longrightarrow \tt ( \sqrt{6} x - \sqrt{3} )( \sqrt{6} x + \sqrt{3} ) = 0[/tex]

either [tex] \tt \sqrt{6} x - \sqrt{3} = 0 \: or \: \sqrt{6} x + \sqrt{3} = 0[/tex]

[tex] \implies \tt \sqrt{6} x = \sqrt{3 \: } \: or \: \: \sqrt{6} x = - \sqrt{3}[/tex]

[tex] \implies \tt x = \dfrac{ \sqrt{3} }{ \sqrt{6} } \: or \: x = - \dfrac{ \sqrt{3} }{ \sqrt{6} }[/tex]

[tex] \implies \tt x = \dfrac{ \sqrt{3} }{ \sqrt{2 \times 3} } \: or \: x = - \dfrac{ \sqrt{3} }{ \sqrt{2 \times 3} }[/tex]

[tex]\implies \tt x = \dfrac{ \cancel{ \sqrt{3} }}{ \sqrt{2} \: \cancel{\sqrt{3}} } \: or \: x = - \dfrac{ \cancel{ \sqrt{3} }}{ \sqrt{2} \: \cancel{\sqrt{3}} }[/tex]

[tex]\implies \tt x = \dfrac{1}{ \sqrt{2} } \: \: or \: \: - \dfrac{1}{ \sqrt{2} }[/tex]

Hence, the zeroes of f(x) = 6x² - 3 are:

[tex] \tt \alpha =\sf \boxed {{ \red{ \dfrac{1}{ \sqrt{2} } } }}\: \: and \: \: \beta =\sf \boxed {{ \red{ - \dfrac{1}{ \sqrt{2} } } }}[/tex]

Verification

Sum of zeroes = [tex] ( \alpha + \beta )[/tex]

[tex] = \tt \dfrac{1}{ \sqrt{2} } + \bigg(- \dfrac{1}{ \sqrt{2} } \bigg)[/tex]

[tex] = \tt \dfrac{1}{ \sqrt{2} } + - \dfrac{1}{ \sqrt{2} } [/tex]

[tex]= \tt 0[/tex]

and, [tex]\tt - \dfrac{Coefficient \: of \: x}{Coefficient \: of \: {x}^{2} }[/tex]

[tex] \tt = - \dfrac{0}{6} [/tex]

[tex] \tt = 0[/tex]

[tex] \therefore \tt \: Sum \: of \: zeroes = {\boxed{ \red{\dfrac{ \tt Coefficient \: of \: x}{ \tt Coefficient \: of \: {x}^{2}}}}}[/tex]

Also,

Product of zeroes = [tex] \alpha \beta [/tex]

[tex] = \dfrac{1}{ \sqrt{2} } \times - \dfrac{1}{ \sqrt{2} } [/tex]

[tex] = - \dfrac{1}{ 2 } [/tex]

and, [tex]\tt - \dfrac{Constant \: term}{Coefficient \: of \: {x}^{2} }[/tex]

[tex] \tt = \dfrac{ - 3}{6} [/tex]

[tex] \tt = \dfrac{ - 1}{2} [/tex]

[tex] \therefore \tt \: Product \: of \: zeroes = {\boxed{ \red{\dfrac{ \tt Constant \: term}{ \tt Coefficient \: of \: {x}^{2}}}}}[/tex]

[tex]\rule{280pt}{2pt}[/tex]

Karl, Serene and Pauline have 1281 stickers altogether. 08
Pauline had 3 times as many stickers as Serene.
Karl had 175 stickers fewer than Pauline.
How many stickers did Pauline have?

Answers

Pauline has 371 stickers. Karl, Serene, and Pauline have 1281 stickers altogether.

What is the equation?

A mathematical statement consisting of an equal symbol between two algebraic expressions with the same value is known as an equation.

Let Karl, Serene and Pauline have x, y, and z stickers.

Given condition;

Karl, Serene, and Pauline have 1281 stickers altogether;

x+y+z = 1281

Pauline had 3 times as many stickers as Serene.

z = 3 y

y=z/3

Karl had 175 stickers fewer than Pauline.

x=z -175

Substitute all the values in the terms of z;

x+y+z = 1281

z -175 + z/3 + z = 1281

8z/3 = 1281 +175

z=546

Pauline have;

x = z -175

x =546 -175

x = 371

Hence Pauline has 371 stickers.

To learn more, about equations, refer;

https://brainly.com/question/10413253

#SPJ1

URGENT WILL GIVE BRANLIEST IF CORRECT!!! A car is moving at 22.5 m/s, when it begins to accelerate at 1.45 m/s². How much time does it take to travel 60.2 m?
(Unit = s)

Answers

Answer:

2.48 seconds

Step-by-step explanation:

See attached

Evaluate the integral
-7x³
x³ +1
Note: Use an upper-case "C" for the constant of integration.
I
dx

Answers

The value of the integration is

[tex]\int {\frac{-7x^{3} }{x^{3}+1} } \, dx =7x - \frac{7}{3} \ln(x+1) + \frac{7}{6} ln (x^{2} -x+1)-\frac{7\sqrt{3} }{3} tan^{-1} (\frac{2x-1}{\sqrt{3} })+C[/tex], where C is an integrating constant.

We know that finding the area of the curve's undersurface is the process of integration. To do this, cover the area with as many tiny rectangles as possible, then add up their areas. The sum gets closer to a limit that corresponds to the area under a function's curve. Finding an antiderivative of a function is the process of integration. If a function can be integrated and its integral over the domain is finite with the given bounds, then the integration is definite.

Here, we have to determine the value of the given integral

[tex]\int {\frac{-7x^{3} }{x^{3}+1} } \, dx[/tex].

So,

[tex]\int {\frac{-7x^{3} }{x^{3}+1} } \, dx = 7 \int {\frac{1-x^{3}-1 }{x^{3}+1} } \, dx[/tex][tex]= 7 \int [{1-\frac{1 }{x^{3}+1} } ]\, dx =7\int \, dx - 7 \int {\frac{1 }{x^{3}+1} } \, dx[/tex] ...(1)

Now,

[tex]\frac{1}{x^{3} +1} = \frac{1}{(x+1)(x^{2} -x+1)}[/tex]

We can write

[tex]\frac{1}{(x+1)(x^{2} -x+1)}=\frac{A}{(x+1)} + \frac{Bx+C}{(x^{2} -x+1)}[/tex]

i.e. [tex]1 = A(x^{2} -x+1)+(Bx+C)(x+1)[/tex]

i.e. [tex]1 = Ax^{2} -Ax + A+Bx^{2} +Bx+Cx+C[/tex]

i.e. [tex]1=(A+B)x^{2} +(-A+B+C)x +(A+C)[/tex]

Comparing both sides, we get

[tex]A+B=0 \implies B = -A[/tex] ...(2)

[tex]-A+B+C = 0 \implies -A+(-A)+C=0 \implies -2A+C=0[/tex] ...(3)

[tex]A+C=1[/tex] ...(4)

Subtracting (3) from (4),

[tex]A+C+2A-C=1-0 \implies 3A=1 \implies A=\frac{1}{3}[/tex]

From (4), [tex]C= 1-\frac{1}{3} =\frac{3-1}{3} =\frac{2}{3}[/tex]

From (2), [tex]B =-\frac{1}{3}[/tex]

We get

[tex]\frac{1}{x^{3}+1 } =\frac{1}{3} \frac{1}{(x+1)} - \frac{1}{3} \frac{x-2}{x^{2} -x+1}[/tex]

[tex]=\frac{1}{3} \frac{1}{(x+1)} - \frac{1}{6} \frac{2x-4}{x^{2} -x+1}[/tex]

[tex]=\frac{1}{3} \frac{1}{(x+1)} - \frac{1}{6} \frac{2x-1-3}{x^{2} -x+1}[/tex]

[tex]=\frac{1}{3} \frac{1}{(x+1)} - \frac{1}{6} \frac{2x-1}{x^{2} -x+1}+\frac{3}{6} \frac{1}{x^{2} -x+1}[/tex]

[tex]=\frac{1}{3} \frac{1}{(x+1)} - \frac{1}{6} \frac{2x-1}{x^{2} -x+1}+\frac{1}{2} \frac{1}{x^{2} -x+1}[/tex]

Integrate both sides,

[tex]\int \frac{1}{x^{3}+1 }\, dx =\frac{1}{3} \int \frac{1}{(x+1)} \, dx - \frac{1}{6} \int \frac{2x-1}{x^{2} -x+1}\, dx+\frac{1}{2} \int \frac{1}{x^{2} -x+1}\, dx[/tex]

i.e. [tex]\int \frac{1}{x^{3}+1 }\, dx =\frac{1}{3} \ln(x+1) - \frac{1}{6} ln (x^{2} -x+1)+\frac{\sqrt{3} }{3} tan^{-1} (\frac{2x-1}{\sqrt{3} })[/tex]

From (1),

[tex]\int {\frac{-7x^{3} }{x^{3}+1} } \, dx =7x - \frac{7}{3} \ln(x+1) + \frac{7}{6} ln (x^{2} -x+1)-\frac{7\sqrt{3} }{3} tan^{-1} (\frac{2x-1}{\sqrt{3} })+C[/tex]

Therefore, the value of the integration is

[tex]\int {\frac{-7x^{3} }{x^{3}+1} } \, dx =7x - \frac{7}{3} \ln(x+1) + \frac{7}{6} ln (x^{2} -x+1)-\frac{7\sqrt{3} }{3} tan^{-1} (\frac{2x-1}{\sqrt{3} })+C[/tex], where C is an integrating constant.

Learn more about integrations here -

https://brainly.com/question/7541827

#SPJ10

Evaluate and reduce:

Answers

The numbers that belongs in the green box=31
-5/8 + 9/2
-5+36/8
31/8
So the exact answer is 31.

After 8 points are added to each score in a sample.
the mean is found to be M = 40. What was the
value for the original mean?

Answers

I think it’s 32 because it’s more simple then it looks! It would just be 40-8??

Fill in the blanks below.
Find the slope of the line passing through the points (8,5) and (8,-4).

Answers

Step-by-step explanation:

please mark me as brainlest

A swimming pool is to be constructed in a 1,408-ft2 backyard. There is to be a fence that will surround a 12-by-24-foot pool. The pool builder wants to build a concrete paver deck of a uniform width, x, surrounding the pool and filling the entire area of the backyard. What is the width of the pool deck?

Answers

The width of the pool is 10 feet.

What is area of rectangle?

The area of rectangle is product of length and breadth.

Let the width be x.

length = 24 + 2x. and breadth =  12 + 2x

We know, area= 1408 ft².

(12 + 2x)(24 + 2x) = 1408

12*24 + 12*2x + 24*2x + (2x)² = 1408

288 + 72x + 4x² = 1408

4x² + 72x + 288 - 1408 = 0

4x² + 72x - 1120 = 0

x² + 18x - 280 = 0

x² - 10x + 28x - 280 = 0

x(x - 10) + 28(x - 10) = 0

(x - 10)(x + 28) = 0

So, x=10, -28.

Hence, the width be 10 feet.

Learn more about this concept here:

https://brainly.com/question/26638297

#SPJ1

Please helpppp I really need please

Answers

Step-by-step explanation:

please mark me as brainlest

Please help whilst I try to solve as well.

Answers

Answer:

Step-by-step explanation:

Answer:

420 cm³

Step-by-step explanation:

we find the volume as if it were a parallelepiped and we remove the volume of the missing part.

8 * 6 * 10 = 480 cm³

480 - 2 * 3 * 10 = 420cm³

what is the answer please?

Answers

Answer:

34.562

Step-by-step explanation:

To find the perimeter of a circle, it's 2 * the radius * pi. In this case, we have a semicircle, so it would be the circle's perimeter divided by 2, or just the radius * pi.

The radius, 11, times pi (3.142), is 34.562.

Brainliest, please :)

The ratio of (1/5 of 245ml) to (0.2 of 0.84l)

Answers

The ratio is given by 98:0.34.

The ratio of (1/5 of 245ml) to (0.2 of 0.84ml).

What is the Ratio?

The ratio can be defined as the comparison of the fraction of one quantity towards others. e.g.- water in milk.

we have  (1/5 of 245ml) to (0.2 of 0.84ml).

⇒ [tex]\frac{1}{5} 245 :0.2*0.84[/tex]

⇒[tex]98:0.34[/tex]

Thus, the required ratio is 98:0.34.

Learn more about Ratio here:
https://brainly.com/question/13419413

#SPJ1

answer the following function, algebra 1.

Answers

Answer:

g(f(x)) = 1x² + 1

Step-by-step explanation:

We are given:
f(x) = x² - 1

g(x) = x + 2

And we want to find g(f(x)).

To do this, we substitute f(x) as x in x+2 (the expression that is equal to g(x)), as g(f(x)) is saying that f(x) is the value of x in g(x).

So, this will be:

g(f(x)) = x² - 1 + 2

Simplify

g(f(x)) = x² + 1

In your system, place 1 in front of x² - this is the coefficient of x², and also 1 after that - this is a constant.

In the equation y/6 = 156, what is the next step in the equation solving sequence

Answers

Answer:

Linear Equations In One Variable =

Next step :

in the equation y/6 = 156, it would be equivalent to y = 156 × 6.

so that :

y = 156 × 6

y = 936

this equation solved. (Answer : 936)

Help please!! I’m stuck with these questions. :)

Answers

Answer:

61

Step-by-step explanation:

the question is asking to find the number of element of set b that are not in set a

to find this one must know the common elements

the question has given the common elements to be 5

now subtract the number of elements in set b with the common elements

i.e. 66-5

=61

Warm Shelters.org received a donation of $32,000 to buy beds and dressers for its new dormitories. Each bed will cost $320.00 and each dresser will be $75.00 each. A total of 132 pieces of beds and dressers are needed. What is the percentage of dressers with respect to the number of beds?

Please HELP

Answers

Answer:

you can buy 98 beds and buy dresses by rest of the money

A company has 200 machines. Each machine has 129 probability of not working. If you were to pick 40 machines randomly, the probability that 5 would not be working is and the probability that at least one machine would be working is the probability that all would be working is

Answers

The probability that 5 would not be working is 0.18665, the probability that at least one machine would be working is 0.00602 and the probability that all would be working is 1.

Given a company has 200 machines. Each machine has a 12% probability of not working.

If we working pick 40 machines randomly then we have to find the probability that 5 would not be working, the probability that at least one machine would be working, and the probability that all would be working.

So

1) probability that 5 would not be working

C(40,5)·0.12⁵·0.88³⁵= 40!/(5!(40-5)!)·0.12⁵·0.88³⁵

                                ≈ 0.18665

2)  probability that at least one machine would be working

0.88⁴⁰ ≈ 0.00602

3) probability that all would be working

1 - 0.12⁴⁰ ≈ 1.0000

Learn more about probability here: https://brainly.com/question/24756209

#SPJ10

As a nurse, part of your daily duties is to mix medications in the proper proportions for your patients. For one of your regular patients, you always mix Medication A with Medication B in the same proportion. Last week, your patient's doctor indicated that you should mix 90 milligrams of Medication A with 72 milligrams of Medication B. However this week, the doctor said to only use 8 milligrams of Medication B. How many milligrams of Medication A should be mixed this week?

Answers

The Medication A mixed this week should be 10 milligrams.

We have,

Medications to be mixed in the proper proportions.

Last week's proportions as the doctor indicated ;

Medication A = 90 milligrams

Medication B = 72 milligrams

This week's proportions as doctor indicated;

Medication B = 8 milligrams

Let, Medication A = x milligrams

As we are instructed to mix the Medication in the proper proportions,

So,

According to the question,

90/72 = x/8

10/8 = x/8

⇒ x = 10 milligrams

Here,  the Medication A mixed is 10 milligrams.

Therefore, the Medication A mixed this week should be 10 milligrams.

Learn more about proportions here:

https://brainly.com/question/7096655

#SPJ10                    

How much will 1,000.00 amount to in 2 years at 13%2% simple interest

Answers

For %13 in 2 years 1,000,000 will amount to 1,260,000

For %2 in 2 years 1,000,000 will amount to 1,040,000

9 is subtracted from 3 times the sum of 4 and 2.​

Answers

Answer:

9

Step-by-step explanation:

This problem is represented below.

=3(4+2)-9

=3(6)-9

=18-9

=9

3 x (4 + 2) - 9 is the representative equation for this sentence. According to pemdas, we first evaluate parenthesis. 4 + 2 = 6. In the next order of pemdas, we multiply. 3 x 6 = 18. Lastly, we can subtract the final two numbers to get our final result for the sentence. 18 - 9 = 9. Therefore, our final answer to this equation is 9.

Your final answer: 3 x (4 + 2) - 9 = 9.

The fox population in a certain region has a continuous growth rate of 7 percent per year. It is estimated that the population in the year 2000 was 15000.

(a) Find a function that models the population
t
years after 2000 (
t
=
0
for 2000).
Your answer is
P
(
t
)
=




(b) Use the function from part (a) to estimate the fox population in the year 2008.
Your answer is (the answer must be an integer)

Answers

There are 25773 foxes in the region in the year 2008

How to determine the function?

The given parameters are:

Initial value, a = 15000Rate, r = 7% or 0.07

Let the number of years after 2000 be x.

So, the function is

f(x) = a * (1 + r)^x

This gives

f(x) = 15000 * (1 + 0.07)^x

In 2008, we have:

x = 8

So, the equation becomes

f(8) = 15000 * (1 + 0.07)^8

Evaluate

f(8) = 25773

Hence, there are 25773 foxes in the region in the year 2008

Read more about exponential functions at:

https://brainly.com/question/2456547

#SPJ1

What is the range of the function in the graph?
. 1≤x≤6
B. 20≤y≤60
C. 20≤x≤60
D. 1≤y≤60

Answers

Answer:

B

Step-by-step explanation:

Range is all the possible values of 'y'. As seen in the graph, the smallest value of 'y' is 20, where the point is (6, 20), and the largest value of 'y' is 60, where the point is (1, 60). Therefore, the range is B. 20≤y≤60

The best possible solution would be B !
Hope this helps :)

Skyler, Robert, and Kaitlyn all solve the same exponential problem but have different approaches. Skyler Robert Kaitlyn 729=9 729=9 729=9 (36)=9 (93)=91 (36)=32 6x = 9 3x = 1 6x = 2 x = 96 x = 13 x = 26 x = 32 x = 13 Explain Sklyer's processes. Is Skyler correct? If not, where did she go wrong? Explain Robert's processes. Is Robert correct? If not, where did he go wrong? Explain Kaitlyn's processes. Is Kaitlyn correct? If not, where did she go wrong?

Answers

The justification for whether Skyler, Robert, or Kaitlyn was right or wrong stems from how they went about their processes:

Skyler erred by failing to equalize the bases of both sides before equating the exponents.Robert is right since he equalized the exponents before comparing the bases of the two sides.Since Kaitlyn made the bases of both sides equal before equating the exponents, she is accurate.

1) SKYLER'S PROCESS:

Skyler wants to resolve the following issue;

Reduce 729 to the lowest base possible as a first step; [3^(6)]^x = 9

729^(x) = 9

The next thing she did was convert 6x to 9.

Since both sides must have the same base when dealing with exponent equality, she may have transformed 9 to have the same base of 3 as the left hand side before equating the exponents.

So Skyler is wrong.

2) ROBERT'S PROCESS:

Skyler failed to capture both bases equally, as we witnessed, but Robert has now made the right decision by;

9^(3x) = 9¹

After which he equated the exponents to get;

3x = 1

x = 1/3

Thus; Robert is correct

3) KAITLYN'S PROCESS:

Kaitlyn is also accurate since she adhered to the same method, which calls for comparing bases before comparing exponents.

Learn more about laws of exponents here:

brainly.com/question/11761858

#SPJ10

the funtions f(x) and g(x) are shown on the graph f(x)=x^2
what is g(x)?

Answers

Answer:

C

Step-by-step explanation:

A reflection of a graph on the x axis simply adds the opposite sign to the x^2, in this case, since f(x) = x^2 is positive, a reflection would mean that g(x) would be -x^2. As g(x) is also vertically translated down 2 units, the equation would then become g(x) = -x^2 - 2 and the correct answer would be C.

Find sin A. 12/13
B. 1
C. 13/12
D. 13/5

Answers

Answer: A. [tex]\frac{12}{13}[/tex]

Step-by-step explanation:

The best way to remember this is SOH-CAH-TOA. For sin, it would be the opposite angle over the hypotenuse. The opposite angle of ∠C is 24 and the hypotenuse is 26. But that isn't in the answer choices, so you'll simplify the fraction.

[tex]\frac{24}{26} =\frac{12}{13}[/tex], therefore, the answer is A.

I hope this helps! Pls mark brainliest!! :)

the angle is 36.2 less than it’s complementary angle

Answers

Answer:

below

Step-by-step explanation:

Measure of each angle is 27 and 63 degrees.

please help! I need this quickly

write an equation in standard form of the line passing through the points (12,6) and (-3,11)​

Answers

Answer:

x + 3y = 30

Step-by-step explanation:

We are given that a line contains the points (12,6) and (-3,11)​.

We want to write the equation of this line in standard form.

Standard form is written as ax+by=c, where a, b, and c are free integer coefficients, however a and b cannot be 0, and a cannot be negative.

Regardless, before we write an equation in slope-intercept form, we must first write the equation in a different form, such as slope-intercept form.

Slope-intercept form is given as y=mx+b, where m is the slope and b is the value of y at the y-intercept.

So first, let's find the slope of the line.
The slope (m) can be found using the formula [tex]\frac{y_2-y_1}{x_2-x_1}[/tex], where [tex](x_1, y_1)[/tex] and [tex](x_2, y_2)[/tex] are points.

Even though we already have 2 points, let's label their values to avoid any confusion and mistakes when calculating.

[tex]x_1=12\\y_1=6\\x_2=-3\\y_2=11[/tex]

Now substitute these values into the formula.

m=[tex]\frac{y_2-y_1}{x_2-x_1}[/tex]

m=[tex]\frac{11-6}{-3-12}[/tex]

Subtract

m=[tex]\frac{5}{-15}[/tex]

Simplify

m=[tex]-\frac{1}{3}[/tex]

The slope is -1/3

Here is the equation of the line so far in slope-intercept form:

[tex]y=-\frac{1}{3} x + b[/tex]

We need to solve for b.

As the equation passes through (12,6) and (-3,11)​, we can use either one to help solve for b.

Taking (12, 6) for example:

[tex]6=-\frac{1}{3}(12) + b[/tex]

Multiply

[tex]6=-\frac{12}{3} + b[/tex]

Divide

6 = -4 + b

Add 4 to both sides.

10 = b

Substitute 10 as b in the equation.

[tex]y = -\frac{1}{3} x + 10[/tex]

Here is the equation in slope-intercept form, but remember, we want it in standard form.

In standard form, the values of both x and y are on the same side, so let's add -1/3x to both sides.

[tex]\frac{1}{3} x + y = 10[/tex]

Remember that a (the coefficient in front of x) has to be an integer, 1/3 is not an integer.

So, let's multiply both sides by 3 to clear the fraction.

[tex]3(\frac{1}{3} x + y) = 3(10)[/tex]
Multiply.

x + 3y = 30

Topic: finding the equation of the line (standard form)

See more: https://brainly.com/question/27575555

If a line has a slope of 4 and goes through the point open parentheses short dash 1 comma 1 close parentheses, then the equation for the line in slope-intercept form is ______________.

a.)
y equals short dash 4 x minus 3

b.)
y equals 4 x plus 3

c.)
y equals 4 x plus 5

d.)
y equals short dash 4 x minus 5

Answers

A line is a one-dimensional shape that is straight, has no thickness, and extends in both directions indefinitely. The equation of the line will be y =4x+5. Thus, the correct option is C.

What is the equation of a line?

A line is a one-dimensional shape that is straight, has no thickness, and extends in both directions indefinitely. The equation of a line is given by,

y =mx + c

where,

x is the coordinate of the x-axis,

y is the coordinate of the y-axis,

m is the slope of the line, and

c is the y-intercept.

Given the slope of the line is 4 and it goes through (-1,1), therefore, the equation of the line will be,

y = mx + c

y = 4x + c

Substitute the value of points,

1 = 4(-1) + c

1 = -4 + c

5 = c

Hence, the equation of the line will be y =4x+5. Thus, the correct option is C.

Learn more about Equation of Line:

https://brainly.com/question/21511618

#SPJ1

Other Questions
The probabilities that Sani, Kolo and Tato will hit a target are 3/4, 2/3 and 7/3 respectively If all three men shot once what is the probability that target will be hit only once which ordered pair is a solution to the following system of inequalities Acontinous random variable X has a pdf given by p(x) = (5x4 0x1 0, otherwise) Let Y=X3. Find the probability distribution function after this i will only have 4 questions please and ty! Which Character is described in these lines "long was the season 12 winters time totures suffered The friend of the Schyldings every affliction endless agony Pams eye-level height is 324 ft above sea level, and adams eye-level height is 400 ft above sea level. how much farther can adam see to the horizon? use the formula d = startroot startfraction 3 h over 2 endfraction endroot, with d being the distance they can see in miles and h being their eye-level height in feet. how do you get your winkey wet?? Tech A says that if a CV boot is torn, it can typically be replaced without replacing the entire joint or shaft. Tech B says that worn CV joints can make a clicking noise when driving while making a tight turn. Who is correct Which expression is equivalent to (3^2) ^-2 nutrients from croplands are carried by the mississippi river into the gulf of mexico. if nutrients are good for growth, why are scientists concerned about this nutrient influx? Which function involves the circulatory system working with the respiratory system?A. Lowering body temperatureB. Enabling cellular respirationC. Stopping bacterial infectionD. Maintaining liver function Neurological research that involves the brain and head size of people with autism strongly suggests that for many Question 21 options: A) their brains are larger than average at birth. B) their brains increase in size after about age five years. C) their brains reach maximum size around age two or three years. D) their brains grow suddenly and excessively in the first two years of lif if three hard drives are using 132 5/8 watts, what is the number of watts used per hard drive?1. 44 7/24 watts2. 42 5/24 watts3. 42 7/24 watts4. 44 5/24 wattswhoever answers correctly i will give brainliest. BRAIN WARM UP MATHS? Tricia recorded the number of pets owned by each of her classmates. these data points represent the results of her survey. 0, 3, 2, 4, 1, 0, 0, 3, 2, 1, 2, 1, 1, 3, 4, 2, 0, 0, 1, 1, 1, 0, 3 create a dot plot that represents the data. a cyclist applies a constant forward force of 20 N to maintain a velocity 2.5 m/s. how much power does the cyclist deliver? Dude this should be so easy but this is going in one ear and out the other how do I do this send help Which atomic models(Bohr, Thomson, Rutherford, Dalton) in task 1 are not supported by Thomsons experimental evidence? For each of these models, explain the experimental results that the model would predict. i need help with this geometry question Which equations have a greater unit rate than the rate represented in the table? x y 3 2 6 4 9 6